You are on page 1of 8

Cinvestav - Autonomous Air and Submarine Navigation Systems

Nonlinear Control, Research PhD. Program


Student: M. Sc. Martı́n Reynoso Cerón
Instructor: PhD. Eduardo Steed Espinoza Quesada

Homework 1
Exercise 1.13
Find a global diffeomorphism z = T (x), that transform the system

ẋ1 = x2 + g1 (x1 )
ẋ2 = x3 + g2 (x1 , x2 )
ẋ3 = g3 (x) + g4 (x)u
y = x1

into
ż1 = z2
ż2 = z3
ż3 = a(z) + b(z)u
y = z1
Assume g1 to g4 are smooth.

Solution

We assume z = T (x) −→ ż = T ẋ, and change of variable as x1 = z1 , x2 = z2 and


x3 = z3 . Then  
T (x) = x2 +gx12(x1 ) x3 +g2x:3
(x1 ,x2 ) 1 (1)

with g3 (x) = a(z) and g4 (x) = b(z)

Exercise 2.1
For each of the following systems

(a) Find all equilibrium points and determine their types

(b)Construc and discuss the phase portrait

(1) ẋ = −x31 + x2 , ẋ2 = x1 + x32


Solution (a)

−x31 + x2 = 0
(2)
x1 + x32 = 0

From −x31 + x2 = 0 −→ x2 = x3 , substituting in downward equation we have

1
x1 (1 + x81 ) = 0. By symmetry in complex plane, the roots of x1 and x2

0 0
   

 1 


 1 

 √1 + √1 i   √1 + √1 i 
 2 2   2 2 
   

 i 


 i 

− √1 + √1 i − √1 + √1 i
x1 =  2 2  , x2 =  2 2 
−1 −1
   
   
 1
− √ − √1 i
 1
− √ − √1 i
 
 2 2   2 2 
−i −i
   
   
1
√ − √ i 1 1
√ − √ i 1
2 2 2 2

Solution (b)

10

5
x2

-5

-10
-10 -8 -6 -4 -2 0 2 4 6 8 10
x1

Figura 1: Phase - Portrait (x1 , x2 ). Exercise (1)

By inspection the 1 the vector field has a saddle points, therefore the system is
unstable.

(3) ẋ1 = x2 , ẋ2 = 2x1 + 3x21 + x31 − x2


Solution (a)

x2 = 0
2x1 + 3x21 + x31 − x2 = 0
It is evident that x2 = 0, 
but 2x1 + 3x21 + x31 − x2 = 0 −→ x1 (x1 + 1)(x1 + 2) = 0 ∴
0
the solutions set of x1 = −1
 
−2
Solution (b)

2
10

5
x2

-5

-10
-10 -8 -6 -4 -2 0 2 4 6 8 10
x1

Figura 2: Phase - Portrait (x1 , x2 ).Exercise (3)

By inspection the 2 the vector field has a stable point in (0, 0), however (0, −1) and
(0, .2) are saddle points, therefore is unstable.

(5) ẋ1 = −x2 , ẋ2 = −x1 + x2 (1 − x21 + 0.1x41 )


Solution (a)

−x2 = 0
−x1 + x2 (1 − x21 + 0.1x41 ) = 0

Evidently x1 = 0 and x2 = 0
Solution (b)

10

5
x2

-5

-10
-10 -8 -6 -4 -2 0 2 4 6 8 10
x1

Figura 3: Phase - Portrait (x1 , x2 ), Exercise (5).

By inspection the the vector field has a saddle point in (0, 0), therefore is unstable.

3
(7) ẋ1 = −x2 , ẋ2 = − (x 1 +x2 )
(x1 +2) (defined over set {x > −2})
Solution (a)

−x2 = 0
− (x 1 +x2 )
(x1 +2) = 0

Evidently x1 = 0 and x2 = 0
Solution (b)

10 x1 ' = x2
x2 ' = - (x1 + x2)/(x1 + 2)

5
x2

-5

-10
-10 -8 -6 -4 -2 0 2 4 6 8 10
x1

Figura 4: Phase - Portrait (x1 , x2 )

By inspection the the vector field has a stable focus in (0, 0), therefore is stable.
1 3
(9) ẋ1 = x2 , ẋ2 = −x1 + 16 x1 + x2 − 31 x32
Solution (a)

−x2 = 0
1 3
−x1 + 16 x1 + x2 − 13 x32 = 0

Evidently x2 = 0 and substituting in downward equation  


0
1 1
then x1 ( 4 x1 + 1)x1 ( 4 x1 − 1) = 0 therefore the solution set for x1 = −4
 
4
Solution (b)

4
10

5
x2

-5

-10
-10 -8 -6 -4 -2 0 2 4 6 8 10
x1

Figura 5: Phase - Portrait (x1 , x2 )

By inspection the 2 the vector field has a stable point in (0, 0), however (0, −4) and
(0, 4) are saddle points, therefore is unstable.

Exercise 2.7
Consider the inverted pendulum equation (A.47) with a = 1

ẋ1 = x2 ; ẋ2 = a(sin x1 + u cos x1 )

(a) With u = 0, find all equilibrium points and determine their types.
Solution
By inspection we have

x1 = nπ
x2 = 0
with n ∈ Z. Then the (nπ, 0) are stable equilibria except (0, 0) that is an unstable
equilibrium point.

(b) The energy E = 1 + cos x1 + 12 x22 is defined such that E = 2 at the upward equili-
brium position (0, 0) and E = 0 at the downward one (π, 0). To swing up to the pen-
dulum we need to pump up the energy. Show that the control u = k(2 − E)x2 cos x1 ,
will make Ė > 0 whenever E < 2 and x2 cos x1 6= 0.
Solution
Tacking into account

ẋ1 = x2 ; ẋ2 = sin x1


in upward position (0, 0) and

ẋ1 = x2 ; ẋ2 = a sin x1 + 2kx2 cos2 x1

5
in downward position (nπ, 0).

The directional time derivative of energy

Ė = x2 ẋ2 − ẋ1 sin x1

(c) Consider the closed-loop system under the foregoing control. Construct and dis-
cuss the phase portrait when k = 0.1
Solution

ẋ1 = x2 ; ẋ2 = sin x1 + 0.2x2 cos2 x1

Solution

10

5
x2

-5

-10
-10 -8 -6 -4 -2 0 2 4 6 8 10
x1

Figura 6: Phase - Portrait (x1 , x2 ) of Pendulum Exercise (c)

By inspection of , it is easily to view the convergence in nπ, 0 focus. Therefore, this


system has multiple equilibria points.

Exercise 3.2
Consider the scalar system ẋ = −g(x), where g(x) is locally Lipschitz and

g(0) = 0; xg(x) > 0, ∀x 6= 0 and x ∈ (−a, a)

It is shown in section 3.1 that the origin is asymptotically stable. In this exercise we arrive
at the same conclusion using Lyapunov functions.

(a) Show that the asymptotically stability conditions of Theorem 3.3 are satisfied
with
Z x
1
V (x) = x2 or V (x) = g(y)dy
2 0

6
(b) If xg(x) > 0 for all x 6= 0, show that the global asymptoticRstability conditions
of Theorem 3.3 are satisfied with V (x) = 12 x2 or V (x) = 21 x2 + 0x g(y)dy.
(c) Under what conditions on g can you show global asymptotic stability using
V (x) = 0x g(y)dy? Give an example of g where the origin of ẋ = −g(x) is globally
R

asymptotic stability conditions of Theorem 3.3.

Exercise 3.3
For each of the following scalar systems, determine if the origin is unstable, stable but
not asymptotically stable but not globally asymptotically stable, or globally asymptotically
stable.
(1) f (x) = −x for x > 0 and f (x) = 0 for x < 0
Solution

(3) f (x) = x for x > 0 and f (x) = 0 for x < 0


(5) sin x

Exercise 3.5
For each the following systems, determine whether the origin is stable, asymptotically
stable or unstable
(1) ẋ1 = x2 ẋ2 = x3 ẋ = − sin x1 − 2x2 − 2x3
Solution
Let the linearization be around of x = 0 as
   
0 1 0 0 1 0
∂f (x)
J(x) = = 0 0 1  = 0 0 1
   
∂x x=0 −2 cos x1 −2 −2 x=0 −2 −2 −2

Next  
−0.2282 + 1.1151i
λi (J(0)) = −0.2282 − 1.1151i −→ Re(λi ) < 0
 
−1.5437
therefore ẋ = f (x) at the origin is exponentially stable, also is stable and asympto-
tically stable
(3) ẋ1 = x2 + x3 ẋ2 = − sin x1 − x3 ẋ3 = sin x1 + x2
Solution
Let the linearization be around of x = 0 as
   
0 1 1 0 1 1
∂f (x)
J(x) = = − cos x1 0 −1 = −1 0 −1
   
∂x x=0
cos x1 1 0 x=0 1 1 0

Next  
−1.0000 + 0.0000i
λi (J(0)) =  0.5000 + 1.3229i 
 
0.5000 − 1.3229i
at least Re(λi ) > 0 therefore ẋ = f (x) at the origin is unstable.

7
Exercise 3.6
Consider the system
ẋ1 = x2
ẋ2 = −h(x1 ) − 2x2 ,
where !
x21
h(x1 ) = x1 2+
1 + x21
verify that V (x) = 0x1 h(σ)dσ + 12 (x1 + x2 )2 is positive define and radially unbounded;
R

then use it to show that is globally exponentially stable.


Solution

V (x) = 0x1 h(σ)dσ + 12 (x1 + x2 )2


R
x1
= σ 2 + 21 σ 2 − 21 log(σ + 1) + 12 (x1 + x2 )2
0
= x21 + 12 x21 − 12 log(x1 + 1) + 21 (x1 + x2 )2
By inspection ∀x 6= 0, V (x) > 0 (positive define). Therefore, V (x) is radially unboun-
ded. it is to say

||x|| −→ ∞ ⇒ V (x) −→ ∞

You might also like